Given that $X,Y$ are independent $N(0,1)$ , show that $frac{XY}{sqrt{X^2+Y^2}},frac{X^2-Y^2}{2sqrt{X^2+Y^2}}$...












5












$begingroup$


It is given that $X,Y overset{text{i.i.d.}}{sim} N(0,1)$



Show that $frac{XY}{sqrt{X^2+Y^2}},frac{X^2-Y^2}{2sqrt{X^2+Y^2}} overset{text{i.i.d.}}{sim} N(0,frac{1}{4})$



I was thinking of making polar transformations $X=r cos theta, Y=r sin theta$



Then I am getting stuck at ranges of $theta$










share|cite|improve this question











$endgroup$












  • $begingroup$
    You're on the right track: don't get discouraged.
    $endgroup$
    – kimchi lover
    Feb 12 '18 at 13:58










  • $begingroup$
    Actually I can show each of the distributions $frac{XY}{sqrt{X^2+Y^2}}$ and the other one to be N(0,1/4) but I cannot show their independence
    $endgroup$
    – Legend Killer
    Feb 12 '18 at 14:00
















5












$begingroup$


It is given that $X,Y overset{text{i.i.d.}}{sim} N(0,1)$



Show that $frac{XY}{sqrt{X^2+Y^2}},frac{X^2-Y^2}{2sqrt{X^2+Y^2}} overset{text{i.i.d.}}{sim} N(0,frac{1}{4})$



I was thinking of making polar transformations $X=r cos theta, Y=r sin theta$



Then I am getting stuck at ranges of $theta$










share|cite|improve this question











$endgroup$












  • $begingroup$
    You're on the right track: don't get discouraged.
    $endgroup$
    – kimchi lover
    Feb 12 '18 at 13:58










  • $begingroup$
    Actually I can show each of the distributions $frac{XY}{sqrt{X^2+Y^2}}$ and the other one to be N(0,1/4) but I cannot show their independence
    $endgroup$
    – Legend Killer
    Feb 12 '18 at 14:00














5












5








5


2



$begingroup$


It is given that $X,Y overset{text{i.i.d.}}{sim} N(0,1)$



Show that $frac{XY}{sqrt{X^2+Y^2}},frac{X^2-Y^2}{2sqrt{X^2+Y^2}} overset{text{i.i.d.}}{sim} N(0,frac{1}{4})$



I was thinking of making polar transformations $X=r cos theta, Y=r sin theta$



Then I am getting stuck at ranges of $theta$










share|cite|improve this question











$endgroup$




It is given that $X,Y overset{text{i.i.d.}}{sim} N(0,1)$



Show that $frac{XY}{sqrt{X^2+Y^2}},frac{X^2-Y^2}{2sqrt{X^2+Y^2}} overset{text{i.i.d.}}{sim} N(0,frac{1}{4})$



I was thinking of making polar transformations $X=r cos theta, Y=r sin theta$



Then I am getting stuck at ranges of $theta$







probability probability-distributions






share|cite|improve this question















share|cite|improve this question













share|cite|improve this question




share|cite|improve this question








edited Aug 20 '18 at 20:40









Michael Hardy

1




1










asked Feb 12 '18 at 13:55









Legend KillerLegend Killer

1,6061523




1,6061523












  • $begingroup$
    You're on the right track: don't get discouraged.
    $endgroup$
    – kimchi lover
    Feb 12 '18 at 13:58










  • $begingroup$
    Actually I can show each of the distributions $frac{XY}{sqrt{X^2+Y^2}}$ and the other one to be N(0,1/4) but I cannot show their independence
    $endgroup$
    – Legend Killer
    Feb 12 '18 at 14:00


















  • $begingroup$
    You're on the right track: don't get discouraged.
    $endgroup$
    – kimchi lover
    Feb 12 '18 at 13:58










  • $begingroup$
    Actually I can show each of the distributions $frac{XY}{sqrt{X^2+Y^2}}$ and the other one to be N(0,1/4) but I cannot show their independence
    $endgroup$
    – Legend Killer
    Feb 12 '18 at 14:00
















$begingroup$
You're on the right track: don't get discouraged.
$endgroup$
– kimchi lover
Feb 12 '18 at 13:58




$begingroup$
You're on the right track: don't get discouraged.
$endgroup$
– kimchi lover
Feb 12 '18 at 13:58












$begingroup$
Actually I can show each of the distributions $frac{XY}{sqrt{X^2+Y^2}}$ and the other one to be N(0,1/4) but I cannot show their independence
$endgroup$
– Legend Killer
Feb 12 '18 at 14:00




$begingroup$
Actually I can show each of the distributions $frac{XY}{sqrt{X^2+Y^2}}$ and the other one to be N(0,1/4) but I cannot show their independence
$endgroup$
– Legend Killer
Feb 12 '18 at 14:00










3 Answers
3






active

oldest

votes


















4












$begingroup$

If you transform $(X,Y)mapsto(R,Theta)$ where $X=RcosTheta,Y=RsinTheta$,



you should end up with the joint density of $(R,Theta)$ as $f_{R,Theta}(r,theta)=dfrac{r}{2pi}e^{-r^2/2}mathbf1_{{r>0,,0<theta<2pi}}$.



This implies $R$ and $Theta$ are independent, where $R$ has the Rayleigh distribution and $Thetasimmathcal{U}(0,2pi)$.



Now changing variables $(R,Theta)mapsto(U,V)$ such that $U=Rsin(2Theta),V=Rcos(2Theta)$,



you should be able to show that $U$ and $V$ are independent $mathcal{N}(0,1)$ variables.



Note that $U=dfrac{2XY}{sqrt{X^2+Y^2}}$ and $V=dfrac{X^2-Y^2}{sqrt{X^2+Y^2}}$ are independent, which in turn means that



$dfrac{U}{2}=dfrac{XY}{sqrt{X^2+Y^2}}$ and $dfrac{V}{2}=dfrac{X^2-Y^2}{2sqrt{X^2+Y^2}}$ are independent $mathcal{N}(0,1/4)$ variables.





This is independent of the post above:



Joint density of $(X,Y)$ is $displaystyle f_{X,Y}(x,y)=frac{1}{2pi}e^{-frac{1}{2}(x^2+y^2)},quad,(x,y)inmathbb{R^2}$



We transform $(X,Y)mapsto(R,Theta)mapsto(U,V)$ where



$x=rcostheta,,y=rsintheta$ and $u=frac{r}{2}sin(2theta),,v=frac{r}{2}cos(2theta)$



$(x,y)inmathbb{R^2}implies r>0,, 0<theta<2piimplies (u,v)inmathbb{R^2}$.



Note that this transformation is not one to one.



Jacobian of the transformation is $Jleft(frac{x,y}{u,v}right) = Jleft(frac{x,y}{r,theta}right)Jleft(frac{r,theta}{u,v}right)=J_1J_2$, say.



Also, $x^2+y^2=r^2=4(u^2+v^2)$ and $|J_1||J_2|=rtimesfrac{2}{r}=2$



Now $left(U=frac{XY}{sqrt{X^2+Y^2}},V=frac{X^2-Y^2}{2sqrt{X^2+Y^2}}right)$ has the preimages $(X,Y)$ and $(-X,-Y)$.



Moreover, $X,Ystackrel{text{i.i.d.}}{sim}mathcal{N}(0,1)iff -X,-Ystackrel{text{i.i.d.}}{sim}mathcal{N}(0,1)$.



Hence the joint density of $(U,V)$ is given by $$f_{U,V}(u,v)=f_{X,Y}(g_1(u,v),h_1(u,v))|J_1||J_2| + f_{X,Y}(g_2(u,v),h_2(u,v))|J_1||J_2|$$



$$=frac{1}{2pi}e^{-frac{1}{2} 4(u^2+v^2)}|J_1||J_2|times 2$$



$$=frac{1}{sqrt{frac{1}{4}}sqrt{2pi}}expleft(-frac{u^2}{2cdotfrac{1}{4}}right)cdotfrac{1}{sqrt{frac{1}{4}}sqrt{2pi}}expleft(-frac{v^2}{2cdotfrac{1}{4}}right)quad ,(u,v)inmathbb{R^2}$$



(We have the multiplier $2$ in the second step due to the preimages of $(x,y)$ namely $(g_i(u,v),h_i(u,v))$ for $i=1,2$, 'contributing' equally to the joint density).



This implies $U$ and $V$ are independent $mathcal{N}(0,1/4)$ variables.






share|cite|improve this answer











$endgroup$













  • $begingroup$
    Yes,I am facing difficulty in showing $U,V $ are N(0,1)
    $endgroup$
    – Legend Killer
    Feb 12 '18 at 14:28










  • $begingroup$
    Ok done! Good solution!
    $endgroup$
    – Legend Killer
    Feb 12 '18 at 14:39










  • $begingroup$
    What about the preimages (u, -v) and (-u,v)?
    $endgroup$
    – Legend Killer
    Feb 24 '18 at 2:13










  • $begingroup$
    @AyanBiswas Let me modify my answer to make it clear.
    $endgroup$
    – StubbornAtom
    Feb 24 '18 at 6:36










  • $begingroup$
    Hi @StubbornAtom. Thank you for this link. Can you please explain as to why the transformation (x,y) -> (u,v) is not one-to-one ? Also, we are mapping from X and Y. Why have you considered -X and -Y here ?
    $endgroup$
    – Dwaipayan Gupta
    Mar 23 '18 at 19:24



















2












$begingroup$

Setting $U=dfrac{XY}{sqrt{X^2+Y^2}}, V=dfrac{X^2-Y^2}{2sqrt{X^2+Y^2}}$ and using moment generating functions:

begin{align}
M_{(U,V)}(u,v) & =mathbb{E}left[mathrm{e}^{langle,(u,v),;,(U,V),rangle}right] \[10pt]
& =iint_{mathbb{R}^2}expleft(ufrac{xy}{sqrt{x^2+y^2}}+vfrac{x^2-y^2}{2sqrt{x^2+y^2}}right)cdot f_{(X,Y)}(x,y),mathrm{d}x,mathrm{d}y.
end{align}
You can see here that $$M_{(U,V)}(u,v)=expleft(frac18(u^2+v^2)right)=expleft(frac12(u,v)
begin{pmatrix}1/4 & 0\ 0 & 1/4end{pmatrix}
(u,v)^Tright)$$

so that $(U,V)$ is normal multivariate with $mathbf{0}$ mean and diagonal covariance matrix, meaning $U, V$ are independant $mathcal{N}(0,1/4)$.






share|cite|improve this answer











$endgroup$





















    2












    $begingroup$

    If you have already proved $frac{X Y}{sqrt{X^2 + Y^2}} $ and $frac{X^2 - Y^2}{sqrt{X^2 + Y^2}}$ are gaussian, as long as that pair of them is jointly gaussian, then you may use their property: $u,, v text{ independent} iff operatorname{cov}(u,v) = 0$.



    $$ operatorname{cov} left( frac{X Y}{sqrt{X^2 + Y^2}},frac{X^2 - Y^2}{sqrt{X^2 + Y^2}} right)$$
    $$ = operatorname{cov} left( frac{X Y}{sqrt{X^2 + Y^2}},frac{X^2}{sqrt{X^2 + Y^2}} right) - operatorname{cov} left( frac{X Y}{sqrt{X^2 + Y^2}},frac{Y^2}{sqrt{X^2 + Y^2}} right)$$



    Now make use of the symmetry of the expressions we have:
    $$ operatorname{cov}(cdots) = 0 $$



    To make this more rigorous one may rewrite the covariance as follows:
    $$ operatorname{cov} left( frac{X Y}{sqrt{X^2 + Y^2}},frac{X^2}{sqrt{X^2 + Y^2}} right) - operatorname{cov} left( frac{X Y}{sqrt{X^2 + Y^2}},frac{Y^2}{sqrt{X^2 + Y^2}} right) $$
    $$ = E left( frac{X^3 Y}{X^2 + Y^2} right) - E left( frac{X Y^3}{X^2 + Y^2} right) $$
    Now renaming $X to Y, , Y to X$ under the first expectation sign (note that this is the same as renaming variables under integral) we get the result.



    Covariance method can be carried even further, following the last line and the trick $ X = -X $ ($X$ is symmetric distribution) which we apply to the second $E$:
    $$ = E left( frac{X^3 Y}{X^2 + Y^2} right) - E left( frac{X Y^3}{X^2 + Y^2} right) = E left( frac{X^3 Y}{X^2 + Y^2} right) + E left( frac{X Y^3}{X^2 + Y^2} right) $$
    $$ = E left( frac{X Y(X^2 + Y^2)}{X^2 + Y^2} right) = E(XY) = 0$$



    The last equation is due to independence of $X, Y$ $implies 0=operatorname{cov}(X,Y)=E(XY) - E(X)E(Y)=E(XY).$






    share|cite|improve this answer











    $endgroup$









    • 3




      $begingroup$
      There is an error in the first sentence here. Being Gaussian and having zero covariance does NOT imply independnce. It does imply independence if they are JOINTLY Gaussian. But you haven't mentioned that.
      $endgroup$
      – Michael Hardy
      Aug 20 '18 at 20:43











    Your Answer





    StackExchange.ifUsing("editor", function () {
    return StackExchange.using("mathjaxEditing", function () {
    StackExchange.MarkdownEditor.creationCallbacks.add(function (editor, postfix) {
    StackExchange.mathjaxEditing.prepareWmdForMathJax(editor, postfix, [["$", "$"], ["\\(","\\)"]]);
    });
    });
    }, "mathjax-editing");

    StackExchange.ready(function() {
    var channelOptions = {
    tags: "".split(" "),
    id: "69"
    };
    initTagRenderer("".split(" "), "".split(" "), channelOptions);

    StackExchange.using("externalEditor", function() {
    // Have to fire editor after snippets, if snippets enabled
    if (StackExchange.settings.snippets.snippetsEnabled) {
    StackExchange.using("snippets", function() {
    createEditor();
    });
    }
    else {
    createEditor();
    }
    });

    function createEditor() {
    StackExchange.prepareEditor({
    heartbeatType: 'answer',
    autoActivateHeartbeat: false,
    convertImagesToLinks: true,
    noModals: true,
    showLowRepImageUploadWarning: true,
    reputationToPostImages: 10,
    bindNavPrevention: true,
    postfix: "",
    imageUploader: {
    brandingHtml: "Powered by u003ca class="icon-imgur-white" href="https://imgur.com/"u003eu003c/au003e",
    contentPolicyHtml: "User contributions licensed under u003ca href="https://creativecommons.org/licenses/by-sa/3.0/"u003ecc by-sa 3.0 with attribution requiredu003c/au003e u003ca href="https://stackoverflow.com/legal/content-policy"u003e(content policy)u003c/au003e",
    allowUrls: true
    },
    noCode: true, onDemand: true,
    discardSelector: ".discard-answer"
    ,immediatelyShowMarkdownHelp:true
    });


    }
    });














    draft saved

    draft discarded


















    StackExchange.ready(
    function () {
    StackExchange.openid.initPostLogin('.new-post-login', 'https%3a%2f%2fmath.stackexchange.com%2fquestions%2f2647442%2fgiven-that-x-y-are-independent-n0-1-show-that-fracxy-sqrtx2y2%23new-answer', 'question_page');
    }
    );

    Post as a guest















    Required, but never shown

























    3 Answers
    3






    active

    oldest

    votes








    3 Answers
    3






    active

    oldest

    votes









    active

    oldest

    votes






    active

    oldest

    votes









    4












    $begingroup$

    If you transform $(X,Y)mapsto(R,Theta)$ where $X=RcosTheta,Y=RsinTheta$,



    you should end up with the joint density of $(R,Theta)$ as $f_{R,Theta}(r,theta)=dfrac{r}{2pi}e^{-r^2/2}mathbf1_{{r>0,,0<theta<2pi}}$.



    This implies $R$ and $Theta$ are independent, where $R$ has the Rayleigh distribution and $Thetasimmathcal{U}(0,2pi)$.



    Now changing variables $(R,Theta)mapsto(U,V)$ such that $U=Rsin(2Theta),V=Rcos(2Theta)$,



    you should be able to show that $U$ and $V$ are independent $mathcal{N}(0,1)$ variables.



    Note that $U=dfrac{2XY}{sqrt{X^2+Y^2}}$ and $V=dfrac{X^2-Y^2}{sqrt{X^2+Y^2}}$ are independent, which in turn means that



    $dfrac{U}{2}=dfrac{XY}{sqrt{X^2+Y^2}}$ and $dfrac{V}{2}=dfrac{X^2-Y^2}{2sqrt{X^2+Y^2}}$ are independent $mathcal{N}(0,1/4)$ variables.





    This is independent of the post above:



    Joint density of $(X,Y)$ is $displaystyle f_{X,Y}(x,y)=frac{1}{2pi}e^{-frac{1}{2}(x^2+y^2)},quad,(x,y)inmathbb{R^2}$



    We transform $(X,Y)mapsto(R,Theta)mapsto(U,V)$ where



    $x=rcostheta,,y=rsintheta$ and $u=frac{r}{2}sin(2theta),,v=frac{r}{2}cos(2theta)$



    $(x,y)inmathbb{R^2}implies r>0,, 0<theta<2piimplies (u,v)inmathbb{R^2}$.



    Note that this transformation is not one to one.



    Jacobian of the transformation is $Jleft(frac{x,y}{u,v}right) = Jleft(frac{x,y}{r,theta}right)Jleft(frac{r,theta}{u,v}right)=J_1J_2$, say.



    Also, $x^2+y^2=r^2=4(u^2+v^2)$ and $|J_1||J_2|=rtimesfrac{2}{r}=2$



    Now $left(U=frac{XY}{sqrt{X^2+Y^2}},V=frac{X^2-Y^2}{2sqrt{X^2+Y^2}}right)$ has the preimages $(X,Y)$ and $(-X,-Y)$.



    Moreover, $X,Ystackrel{text{i.i.d.}}{sim}mathcal{N}(0,1)iff -X,-Ystackrel{text{i.i.d.}}{sim}mathcal{N}(0,1)$.



    Hence the joint density of $(U,V)$ is given by $$f_{U,V}(u,v)=f_{X,Y}(g_1(u,v),h_1(u,v))|J_1||J_2| + f_{X,Y}(g_2(u,v),h_2(u,v))|J_1||J_2|$$



    $$=frac{1}{2pi}e^{-frac{1}{2} 4(u^2+v^2)}|J_1||J_2|times 2$$



    $$=frac{1}{sqrt{frac{1}{4}}sqrt{2pi}}expleft(-frac{u^2}{2cdotfrac{1}{4}}right)cdotfrac{1}{sqrt{frac{1}{4}}sqrt{2pi}}expleft(-frac{v^2}{2cdotfrac{1}{4}}right)quad ,(u,v)inmathbb{R^2}$$



    (We have the multiplier $2$ in the second step due to the preimages of $(x,y)$ namely $(g_i(u,v),h_i(u,v))$ for $i=1,2$, 'contributing' equally to the joint density).



    This implies $U$ and $V$ are independent $mathcal{N}(0,1/4)$ variables.






    share|cite|improve this answer











    $endgroup$













    • $begingroup$
      Yes,I am facing difficulty in showing $U,V $ are N(0,1)
      $endgroup$
      – Legend Killer
      Feb 12 '18 at 14:28










    • $begingroup$
      Ok done! Good solution!
      $endgroup$
      – Legend Killer
      Feb 12 '18 at 14:39










    • $begingroup$
      What about the preimages (u, -v) and (-u,v)?
      $endgroup$
      – Legend Killer
      Feb 24 '18 at 2:13










    • $begingroup$
      @AyanBiswas Let me modify my answer to make it clear.
      $endgroup$
      – StubbornAtom
      Feb 24 '18 at 6:36










    • $begingroup$
      Hi @StubbornAtom. Thank you for this link. Can you please explain as to why the transformation (x,y) -> (u,v) is not one-to-one ? Also, we are mapping from X and Y. Why have you considered -X and -Y here ?
      $endgroup$
      – Dwaipayan Gupta
      Mar 23 '18 at 19:24
















    4












    $begingroup$

    If you transform $(X,Y)mapsto(R,Theta)$ where $X=RcosTheta,Y=RsinTheta$,



    you should end up with the joint density of $(R,Theta)$ as $f_{R,Theta}(r,theta)=dfrac{r}{2pi}e^{-r^2/2}mathbf1_{{r>0,,0<theta<2pi}}$.



    This implies $R$ and $Theta$ are independent, where $R$ has the Rayleigh distribution and $Thetasimmathcal{U}(0,2pi)$.



    Now changing variables $(R,Theta)mapsto(U,V)$ such that $U=Rsin(2Theta),V=Rcos(2Theta)$,



    you should be able to show that $U$ and $V$ are independent $mathcal{N}(0,1)$ variables.



    Note that $U=dfrac{2XY}{sqrt{X^2+Y^2}}$ and $V=dfrac{X^2-Y^2}{sqrt{X^2+Y^2}}$ are independent, which in turn means that



    $dfrac{U}{2}=dfrac{XY}{sqrt{X^2+Y^2}}$ and $dfrac{V}{2}=dfrac{X^2-Y^2}{2sqrt{X^2+Y^2}}$ are independent $mathcal{N}(0,1/4)$ variables.





    This is independent of the post above:



    Joint density of $(X,Y)$ is $displaystyle f_{X,Y}(x,y)=frac{1}{2pi}e^{-frac{1}{2}(x^2+y^2)},quad,(x,y)inmathbb{R^2}$



    We transform $(X,Y)mapsto(R,Theta)mapsto(U,V)$ where



    $x=rcostheta,,y=rsintheta$ and $u=frac{r}{2}sin(2theta),,v=frac{r}{2}cos(2theta)$



    $(x,y)inmathbb{R^2}implies r>0,, 0<theta<2piimplies (u,v)inmathbb{R^2}$.



    Note that this transformation is not one to one.



    Jacobian of the transformation is $Jleft(frac{x,y}{u,v}right) = Jleft(frac{x,y}{r,theta}right)Jleft(frac{r,theta}{u,v}right)=J_1J_2$, say.



    Also, $x^2+y^2=r^2=4(u^2+v^2)$ and $|J_1||J_2|=rtimesfrac{2}{r}=2$



    Now $left(U=frac{XY}{sqrt{X^2+Y^2}},V=frac{X^2-Y^2}{2sqrt{X^2+Y^2}}right)$ has the preimages $(X,Y)$ and $(-X,-Y)$.



    Moreover, $X,Ystackrel{text{i.i.d.}}{sim}mathcal{N}(0,1)iff -X,-Ystackrel{text{i.i.d.}}{sim}mathcal{N}(0,1)$.



    Hence the joint density of $(U,V)$ is given by $$f_{U,V}(u,v)=f_{X,Y}(g_1(u,v),h_1(u,v))|J_1||J_2| + f_{X,Y}(g_2(u,v),h_2(u,v))|J_1||J_2|$$



    $$=frac{1}{2pi}e^{-frac{1}{2} 4(u^2+v^2)}|J_1||J_2|times 2$$



    $$=frac{1}{sqrt{frac{1}{4}}sqrt{2pi}}expleft(-frac{u^2}{2cdotfrac{1}{4}}right)cdotfrac{1}{sqrt{frac{1}{4}}sqrt{2pi}}expleft(-frac{v^2}{2cdotfrac{1}{4}}right)quad ,(u,v)inmathbb{R^2}$$



    (We have the multiplier $2$ in the second step due to the preimages of $(x,y)$ namely $(g_i(u,v),h_i(u,v))$ for $i=1,2$, 'contributing' equally to the joint density).



    This implies $U$ and $V$ are independent $mathcal{N}(0,1/4)$ variables.






    share|cite|improve this answer











    $endgroup$













    • $begingroup$
      Yes,I am facing difficulty in showing $U,V $ are N(0,1)
      $endgroup$
      – Legend Killer
      Feb 12 '18 at 14:28










    • $begingroup$
      Ok done! Good solution!
      $endgroup$
      – Legend Killer
      Feb 12 '18 at 14:39










    • $begingroup$
      What about the preimages (u, -v) and (-u,v)?
      $endgroup$
      – Legend Killer
      Feb 24 '18 at 2:13










    • $begingroup$
      @AyanBiswas Let me modify my answer to make it clear.
      $endgroup$
      – StubbornAtom
      Feb 24 '18 at 6:36










    • $begingroup$
      Hi @StubbornAtom. Thank you for this link. Can you please explain as to why the transformation (x,y) -> (u,v) is not one-to-one ? Also, we are mapping from X and Y. Why have you considered -X and -Y here ?
      $endgroup$
      – Dwaipayan Gupta
      Mar 23 '18 at 19:24














    4












    4








    4





    $begingroup$

    If you transform $(X,Y)mapsto(R,Theta)$ where $X=RcosTheta,Y=RsinTheta$,



    you should end up with the joint density of $(R,Theta)$ as $f_{R,Theta}(r,theta)=dfrac{r}{2pi}e^{-r^2/2}mathbf1_{{r>0,,0<theta<2pi}}$.



    This implies $R$ and $Theta$ are independent, where $R$ has the Rayleigh distribution and $Thetasimmathcal{U}(0,2pi)$.



    Now changing variables $(R,Theta)mapsto(U,V)$ such that $U=Rsin(2Theta),V=Rcos(2Theta)$,



    you should be able to show that $U$ and $V$ are independent $mathcal{N}(0,1)$ variables.



    Note that $U=dfrac{2XY}{sqrt{X^2+Y^2}}$ and $V=dfrac{X^2-Y^2}{sqrt{X^2+Y^2}}$ are independent, which in turn means that



    $dfrac{U}{2}=dfrac{XY}{sqrt{X^2+Y^2}}$ and $dfrac{V}{2}=dfrac{X^2-Y^2}{2sqrt{X^2+Y^2}}$ are independent $mathcal{N}(0,1/4)$ variables.





    This is independent of the post above:



    Joint density of $(X,Y)$ is $displaystyle f_{X,Y}(x,y)=frac{1}{2pi}e^{-frac{1}{2}(x^2+y^2)},quad,(x,y)inmathbb{R^2}$



    We transform $(X,Y)mapsto(R,Theta)mapsto(U,V)$ where



    $x=rcostheta,,y=rsintheta$ and $u=frac{r}{2}sin(2theta),,v=frac{r}{2}cos(2theta)$



    $(x,y)inmathbb{R^2}implies r>0,, 0<theta<2piimplies (u,v)inmathbb{R^2}$.



    Note that this transformation is not one to one.



    Jacobian of the transformation is $Jleft(frac{x,y}{u,v}right) = Jleft(frac{x,y}{r,theta}right)Jleft(frac{r,theta}{u,v}right)=J_1J_2$, say.



    Also, $x^2+y^2=r^2=4(u^2+v^2)$ and $|J_1||J_2|=rtimesfrac{2}{r}=2$



    Now $left(U=frac{XY}{sqrt{X^2+Y^2}},V=frac{X^2-Y^2}{2sqrt{X^2+Y^2}}right)$ has the preimages $(X,Y)$ and $(-X,-Y)$.



    Moreover, $X,Ystackrel{text{i.i.d.}}{sim}mathcal{N}(0,1)iff -X,-Ystackrel{text{i.i.d.}}{sim}mathcal{N}(0,1)$.



    Hence the joint density of $(U,V)$ is given by $$f_{U,V}(u,v)=f_{X,Y}(g_1(u,v),h_1(u,v))|J_1||J_2| + f_{X,Y}(g_2(u,v),h_2(u,v))|J_1||J_2|$$



    $$=frac{1}{2pi}e^{-frac{1}{2} 4(u^2+v^2)}|J_1||J_2|times 2$$



    $$=frac{1}{sqrt{frac{1}{4}}sqrt{2pi}}expleft(-frac{u^2}{2cdotfrac{1}{4}}right)cdotfrac{1}{sqrt{frac{1}{4}}sqrt{2pi}}expleft(-frac{v^2}{2cdotfrac{1}{4}}right)quad ,(u,v)inmathbb{R^2}$$



    (We have the multiplier $2$ in the second step due to the preimages of $(x,y)$ namely $(g_i(u,v),h_i(u,v))$ for $i=1,2$, 'contributing' equally to the joint density).



    This implies $U$ and $V$ are independent $mathcal{N}(0,1/4)$ variables.






    share|cite|improve this answer











    $endgroup$



    If you transform $(X,Y)mapsto(R,Theta)$ where $X=RcosTheta,Y=RsinTheta$,



    you should end up with the joint density of $(R,Theta)$ as $f_{R,Theta}(r,theta)=dfrac{r}{2pi}e^{-r^2/2}mathbf1_{{r>0,,0<theta<2pi}}$.



    This implies $R$ and $Theta$ are independent, where $R$ has the Rayleigh distribution and $Thetasimmathcal{U}(0,2pi)$.



    Now changing variables $(R,Theta)mapsto(U,V)$ such that $U=Rsin(2Theta),V=Rcos(2Theta)$,



    you should be able to show that $U$ and $V$ are independent $mathcal{N}(0,1)$ variables.



    Note that $U=dfrac{2XY}{sqrt{X^2+Y^2}}$ and $V=dfrac{X^2-Y^2}{sqrt{X^2+Y^2}}$ are independent, which in turn means that



    $dfrac{U}{2}=dfrac{XY}{sqrt{X^2+Y^2}}$ and $dfrac{V}{2}=dfrac{X^2-Y^2}{2sqrt{X^2+Y^2}}$ are independent $mathcal{N}(0,1/4)$ variables.





    This is independent of the post above:



    Joint density of $(X,Y)$ is $displaystyle f_{X,Y}(x,y)=frac{1}{2pi}e^{-frac{1}{2}(x^2+y^2)},quad,(x,y)inmathbb{R^2}$



    We transform $(X,Y)mapsto(R,Theta)mapsto(U,V)$ where



    $x=rcostheta,,y=rsintheta$ and $u=frac{r}{2}sin(2theta),,v=frac{r}{2}cos(2theta)$



    $(x,y)inmathbb{R^2}implies r>0,, 0<theta<2piimplies (u,v)inmathbb{R^2}$.



    Note that this transformation is not one to one.



    Jacobian of the transformation is $Jleft(frac{x,y}{u,v}right) = Jleft(frac{x,y}{r,theta}right)Jleft(frac{r,theta}{u,v}right)=J_1J_2$, say.



    Also, $x^2+y^2=r^2=4(u^2+v^2)$ and $|J_1||J_2|=rtimesfrac{2}{r}=2$



    Now $left(U=frac{XY}{sqrt{X^2+Y^2}},V=frac{X^2-Y^2}{2sqrt{X^2+Y^2}}right)$ has the preimages $(X,Y)$ and $(-X,-Y)$.



    Moreover, $X,Ystackrel{text{i.i.d.}}{sim}mathcal{N}(0,1)iff -X,-Ystackrel{text{i.i.d.}}{sim}mathcal{N}(0,1)$.



    Hence the joint density of $(U,V)$ is given by $$f_{U,V}(u,v)=f_{X,Y}(g_1(u,v),h_1(u,v))|J_1||J_2| + f_{X,Y}(g_2(u,v),h_2(u,v))|J_1||J_2|$$



    $$=frac{1}{2pi}e^{-frac{1}{2} 4(u^2+v^2)}|J_1||J_2|times 2$$



    $$=frac{1}{sqrt{frac{1}{4}}sqrt{2pi}}expleft(-frac{u^2}{2cdotfrac{1}{4}}right)cdotfrac{1}{sqrt{frac{1}{4}}sqrt{2pi}}expleft(-frac{v^2}{2cdotfrac{1}{4}}right)quad ,(u,v)inmathbb{R^2}$$



    (We have the multiplier $2$ in the second step due to the preimages of $(x,y)$ namely $(g_i(u,v),h_i(u,v))$ for $i=1,2$, 'contributing' equally to the joint density).



    This implies $U$ and $V$ are independent $mathcal{N}(0,1/4)$ variables.







    share|cite|improve this answer














    share|cite|improve this answer



    share|cite|improve this answer








    edited Aug 20 '18 at 20:49









    Michael Hardy

    1




    1










    answered Feb 12 '18 at 14:18









    StubbornAtomStubbornAtom

    5,67411138




    5,67411138












    • $begingroup$
      Yes,I am facing difficulty in showing $U,V $ are N(0,1)
      $endgroup$
      – Legend Killer
      Feb 12 '18 at 14:28










    • $begingroup$
      Ok done! Good solution!
      $endgroup$
      – Legend Killer
      Feb 12 '18 at 14:39










    • $begingroup$
      What about the preimages (u, -v) and (-u,v)?
      $endgroup$
      – Legend Killer
      Feb 24 '18 at 2:13










    • $begingroup$
      @AyanBiswas Let me modify my answer to make it clear.
      $endgroup$
      – StubbornAtom
      Feb 24 '18 at 6:36










    • $begingroup$
      Hi @StubbornAtom. Thank you for this link. Can you please explain as to why the transformation (x,y) -> (u,v) is not one-to-one ? Also, we are mapping from X and Y. Why have you considered -X and -Y here ?
      $endgroup$
      – Dwaipayan Gupta
      Mar 23 '18 at 19:24


















    • $begingroup$
      Yes,I am facing difficulty in showing $U,V $ are N(0,1)
      $endgroup$
      – Legend Killer
      Feb 12 '18 at 14:28










    • $begingroup$
      Ok done! Good solution!
      $endgroup$
      – Legend Killer
      Feb 12 '18 at 14:39










    • $begingroup$
      What about the preimages (u, -v) and (-u,v)?
      $endgroup$
      – Legend Killer
      Feb 24 '18 at 2:13










    • $begingroup$
      @AyanBiswas Let me modify my answer to make it clear.
      $endgroup$
      – StubbornAtom
      Feb 24 '18 at 6:36










    • $begingroup$
      Hi @StubbornAtom. Thank you for this link. Can you please explain as to why the transformation (x,y) -> (u,v) is not one-to-one ? Also, we are mapping from X and Y. Why have you considered -X and -Y here ?
      $endgroup$
      – Dwaipayan Gupta
      Mar 23 '18 at 19:24
















    $begingroup$
    Yes,I am facing difficulty in showing $U,V $ are N(0,1)
    $endgroup$
    – Legend Killer
    Feb 12 '18 at 14:28




    $begingroup$
    Yes,I am facing difficulty in showing $U,V $ are N(0,1)
    $endgroup$
    – Legend Killer
    Feb 12 '18 at 14:28












    $begingroup$
    Ok done! Good solution!
    $endgroup$
    – Legend Killer
    Feb 12 '18 at 14:39




    $begingroup$
    Ok done! Good solution!
    $endgroup$
    – Legend Killer
    Feb 12 '18 at 14:39












    $begingroup$
    What about the preimages (u, -v) and (-u,v)?
    $endgroup$
    – Legend Killer
    Feb 24 '18 at 2:13




    $begingroup$
    What about the preimages (u, -v) and (-u,v)?
    $endgroup$
    – Legend Killer
    Feb 24 '18 at 2:13












    $begingroup$
    @AyanBiswas Let me modify my answer to make it clear.
    $endgroup$
    – StubbornAtom
    Feb 24 '18 at 6:36




    $begingroup$
    @AyanBiswas Let me modify my answer to make it clear.
    $endgroup$
    – StubbornAtom
    Feb 24 '18 at 6:36












    $begingroup$
    Hi @StubbornAtom. Thank you for this link. Can you please explain as to why the transformation (x,y) -> (u,v) is not one-to-one ? Also, we are mapping from X and Y. Why have you considered -X and -Y here ?
    $endgroup$
    – Dwaipayan Gupta
    Mar 23 '18 at 19:24




    $begingroup$
    Hi @StubbornAtom. Thank you for this link. Can you please explain as to why the transformation (x,y) -> (u,v) is not one-to-one ? Also, we are mapping from X and Y. Why have you considered -X and -Y here ?
    $endgroup$
    – Dwaipayan Gupta
    Mar 23 '18 at 19:24











    2












    $begingroup$

    Setting $U=dfrac{XY}{sqrt{X^2+Y^2}}, V=dfrac{X^2-Y^2}{2sqrt{X^2+Y^2}}$ and using moment generating functions:

    begin{align}
    M_{(U,V)}(u,v) & =mathbb{E}left[mathrm{e}^{langle,(u,v),;,(U,V),rangle}right] \[10pt]
    & =iint_{mathbb{R}^2}expleft(ufrac{xy}{sqrt{x^2+y^2}}+vfrac{x^2-y^2}{2sqrt{x^2+y^2}}right)cdot f_{(X,Y)}(x,y),mathrm{d}x,mathrm{d}y.
    end{align}
    You can see here that $$M_{(U,V)}(u,v)=expleft(frac18(u^2+v^2)right)=expleft(frac12(u,v)
    begin{pmatrix}1/4 & 0\ 0 & 1/4end{pmatrix}
    (u,v)^Tright)$$

    so that $(U,V)$ is normal multivariate with $mathbf{0}$ mean and diagonal covariance matrix, meaning $U, V$ are independant $mathcal{N}(0,1/4)$.






    share|cite|improve this answer











    $endgroup$


















      2












      $begingroup$

      Setting $U=dfrac{XY}{sqrt{X^2+Y^2}}, V=dfrac{X^2-Y^2}{2sqrt{X^2+Y^2}}$ and using moment generating functions:

      begin{align}
      M_{(U,V)}(u,v) & =mathbb{E}left[mathrm{e}^{langle,(u,v),;,(U,V),rangle}right] \[10pt]
      & =iint_{mathbb{R}^2}expleft(ufrac{xy}{sqrt{x^2+y^2}}+vfrac{x^2-y^2}{2sqrt{x^2+y^2}}right)cdot f_{(X,Y)}(x,y),mathrm{d}x,mathrm{d}y.
      end{align}
      You can see here that $$M_{(U,V)}(u,v)=expleft(frac18(u^2+v^2)right)=expleft(frac12(u,v)
      begin{pmatrix}1/4 & 0\ 0 & 1/4end{pmatrix}
      (u,v)^Tright)$$

      so that $(U,V)$ is normal multivariate with $mathbf{0}$ mean and diagonal covariance matrix, meaning $U, V$ are independant $mathcal{N}(0,1/4)$.






      share|cite|improve this answer











      $endgroup$
















        2












        2








        2





        $begingroup$

        Setting $U=dfrac{XY}{sqrt{X^2+Y^2}}, V=dfrac{X^2-Y^2}{2sqrt{X^2+Y^2}}$ and using moment generating functions:

        begin{align}
        M_{(U,V)}(u,v) & =mathbb{E}left[mathrm{e}^{langle,(u,v),;,(U,V),rangle}right] \[10pt]
        & =iint_{mathbb{R}^2}expleft(ufrac{xy}{sqrt{x^2+y^2}}+vfrac{x^2-y^2}{2sqrt{x^2+y^2}}right)cdot f_{(X,Y)}(x,y),mathrm{d}x,mathrm{d}y.
        end{align}
        You can see here that $$M_{(U,V)}(u,v)=expleft(frac18(u^2+v^2)right)=expleft(frac12(u,v)
        begin{pmatrix}1/4 & 0\ 0 & 1/4end{pmatrix}
        (u,v)^Tright)$$

        so that $(U,V)$ is normal multivariate with $mathbf{0}$ mean and diagonal covariance matrix, meaning $U, V$ are independant $mathcal{N}(0,1/4)$.






        share|cite|improve this answer











        $endgroup$



        Setting $U=dfrac{XY}{sqrt{X^2+Y^2}}, V=dfrac{X^2-Y^2}{2sqrt{X^2+Y^2}}$ and using moment generating functions:

        begin{align}
        M_{(U,V)}(u,v) & =mathbb{E}left[mathrm{e}^{langle,(u,v),;,(U,V),rangle}right] \[10pt]
        & =iint_{mathbb{R}^2}expleft(ufrac{xy}{sqrt{x^2+y^2}}+vfrac{x^2-y^2}{2sqrt{x^2+y^2}}right)cdot f_{(X,Y)}(x,y),mathrm{d}x,mathrm{d}y.
        end{align}
        You can see here that $$M_{(U,V)}(u,v)=expleft(frac18(u^2+v^2)right)=expleft(frac12(u,v)
        begin{pmatrix}1/4 & 0\ 0 & 1/4end{pmatrix}
        (u,v)^Tright)$$

        so that $(U,V)$ is normal multivariate with $mathbf{0}$ mean and diagonal covariance matrix, meaning $U, V$ are independant $mathcal{N}(0,1/4)$.







        share|cite|improve this answer














        share|cite|improve this answer



        share|cite|improve this answer








        edited Aug 20 '18 at 20:43









        Michael Hardy

        1




        1










        answered Feb 24 '18 at 21:37









        dequedeque

        413111




        413111























            2












            $begingroup$

            If you have already proved $frac{X Y}{sqrt{X^2 + Y^2}} $ and $frac{X^2 - Y^2}{sqrt{X^2 + Y^2}}$ are gaussian, as long as that pair of them is jointly gaussian, then you may use their property: $u,, v text{ independent} iff operatorname{cov}(u,v) = 0$.



            $$ operatorname{cov} left( frac{X Y}{sqrt{X^2 + Y^2}},frac{X^2 - Y^2}{sqrt{X^2 + Y^2}} right)$$
            $$ = operatorname{cov} left( frac{X Y}{sqrt{X^2 + Y^2}},frac{X^2}{sqrt{X^2 + Y^2}} right) - operatorname{cov} left( frac{X Y}{sqrt{X^2 + Y^2}},frac{Y^2}{sqrt{X^2 + Y^2}} right)$$



            Now make use of the symmetry of the expressions we have:
            $$ operatorname{cov}(cdots) = 0 $$



            To make this more rigorous one may rewrite the covariance as follows:
            $$ operatorname{cov} left( frac{X Y}{sqrt{X^2 + Y^2}},frac{X^2}{sqrt{X^2 + Y^2}} right) - operatorname{cov} left( frac{X Y}{sqrt{X^2 + Y^2}},frac{Y^2}{sqrt{X^2 + Y^2}} right) $$
            $$ = E left( frac{X^3 Y}{X^2 + Y^2} right) - E left( frac{X Y^3}{X^2 + Y^2} right) $$
            Now renaming $X to Y, , Y to X$ under the first expectation sign (note that this is the same as renaming variables under integral) we get the result.



            Covariance method can be carried even further, following the last line and the trick $ X = -X $ ($X$ is symmetric distribution) which we apply to the second $E$:
            $$ = E left( frac{X^3 Y}{X^2 + Y^2} right) - E left( frac{X Y^3}{X^2 + Y^2} right) = E left( frac{X^3 Y}{X^2 + Y^2} right) + E left( frac{X Y^3}{X^2 + Y^2} right) $$
            $$ = E left( frac{X Y(X^2 + Y^2)}{X^2 + Y^2} right) = E(XY) = 0$$



            The last equation is due to independence of $X, Y$ $implies 0=operatorname{cov}(X,Y)=E(XY) - E(X)E(Y)=E(XY).$






            share|cite|improve this answer











            $endgroup$









            • 3




              $begingroup$
              There is an error in the first sentence here. Being Gaussian and having zero covariance does NOT imply independnce. It does imply independence if they are JOINTLY Gaussian. But you haven't mentioned that.
              $endgroup$
              – Michael Hardy
              Aug 20 '18 at 20:43
















            2












            $begingroup$

            If you have already proved $frac{X Y}{sqrt{X^2 + Y^2}} $ and $frac{X^2 - Y^2}{sqrt{X^2 + Y^2}}$ are gaussian, as long as that pair of them is jointly gaussian, then you may use their property: $u,, v text{ independent} iff operatorname{cov}(u,v) = 0$.



            $$ operatorname{cov} left( frac{X Y}{sqrt{X^2 + Y^2}},frac{X^2 - Y^2}{sqrt{X^2 + Y^2}} right)$$
            $$ = operatorname{cov} left( frac{X Y}{sqrt{X^2 + Y^2}},frac{X^2}{sqrt{X^2 + Y^2}} right) - operatorname{cov} left( frac{X Y}{sqrt{X^2 + Y^2}},frac{Y^2}{sqrt{X^2 + Y^2}} right)$$



            Now make use of the symmetry of the expressions we have:
            $$ operatorname{cov}(cdots) = 0 $$



            To make this more rigorous one may rewrite the covariance as follows:
            $$ operatorname{cov} left( frac{X Y}{sqrt{X^2 + Y^2}},frac{X^2}{sqrt{X^2 + Y^2}} right) - operatorname{cov} left( frac{X Y}{sqrt{X^2 + Y^2}},frac{Y^2}{sqrt{X^2 + Y^2}} right) $$
            $$ = E left( frac{X^3 Y}{X^2 + Y^2} right) - E left( frac{X Y^3}{X^2 + Y^2} right) $$
            Now renaming $X to Y, , Y to X$ under the first expectation sign (note that this is the same as renaming variables under integral) we get the result.



            Covariance method can be carried even further, following the last line and the trick $ X = -X $ ($X$ is symmetric distribution) which we apply to the second $E$:
            $$ = E left( frac{X^3 Y}{X^2 + Y^2} right) - E left( frac{X Y^3}{X^2 + Y^2} right) = E left( frac{X^3 Y}{X^2 + Y^2} right) + E left( frac{X Y^3}{X^2 + Y^2} right) $$
            $$ = E left( frac{X Y(X^2 + Y^2)}{X^2 + Y^2} right) = E(XY) = 0$$



            The last equation is due to independence of $X, Y$ $implies 0=operatorname{cov}(X,Y)=E(XY) - E(X)E(Y)=E(XY).$






            share|cite|improve this answer











            $endgroup$









            • 3




              $begingroup$
              There is an error in the first sentence here. Being Gaussian and having zero covariance does NOT imply independnce. It does imply independence if they are JOINTLY Gaussian. But you haven't mentioned that.
              $endgroup$
              – Michael Hardy
              Aug 20 '18 at 20:43














            2












            2








            2





            $begingroup$

            If you have already proved $frac{X Y}{sqrt{X^2 + Y^2}} $ and $frac{X^2 - Y^2}{sqrt{X^2 + Y^2}}$ are gaussian, as long as that pair of them is jointly gaussian, then you may use their property: $u,, v text{ independent} iff operatorname{cov}(u,v) = 0$.



            $$ operatorname{cov} left( frac{X Y}{sqrt{X^2 + Y^2}},frac{X^2 - Y^2}{sqrt{X^2 + Y^2}} right)$$
            $$ = operatorname{cov} left( frac{X Y}{sqrt{X^2 + Y^2}},frac{X^2}{sqrt{X^2 + Y^2}} right) - operatorname{cov} left( frac{X Y}{sqrt{X^2 + Y^2}},frac{Y^2}{sqrt{X^2 + Y^2}} right)$$



            Now make use of the symmetry of the expressions we have:
            $$ operatorname{cov}(cdots) = 0 $$



            To make this more rigorous one may rewrite the covariance as follows:
            $$ operatorname{cov} left( frac{X Y}{sqrt{X^2 + Y^2}},frac{X^2}{sqrt{X^2 + Y^2}} right) - operatorname{cov} left( frac{X Y}{sqrt{X^2 + Y^2}},frac{Y^2}{sqrt{X^2 + Y^2}} right) $$
            $$ = E left( frac{X^3 Y}{X^2 + Y^2} right) - E left( frac{X Y^3}{X^2 + Y^2} right) $$
            Now renaming $X to Y, , Y to X$ under the first expectation sign (note that this is the same as renaming variables under integral) we get the result.



            Covariance method can be carried even further, following the last line and the trick $ X = -X $ ($X$ is symmetric distribution) which we apply to the second $E$:
            $$ = E left( frac{X^3 Y}{X^2 + Y^2} right) - E left( frac{X Y^3}{X^2 + Y^2} right) = E left( frac{X^3 Y}{X^2 + Y^2} right) + E left( frac{X Y^3}{X^2 + Y^2} right) $$
            $$ = E left( frac{X Y(X^2 + Y^2)}{X^2 + Y^2} right) = E(XY) = 0$$



            The last equation is due to independence of $X, Y$ $implies 0=operatorname{cov}(X,Y)=E(XY) - E(X)E(Y)=E(XY).$






            share|cite|improve this answer











            $endgroup$



            If you have already proved $frac{X Y}{sqrt{X^2 + Y^2}} $ and $frac{X^2 - Y^2}{sqrt{X^2 + Y^2}}$ are gaussian, as long as that pair of them is jointly gaussian, then you may use their property: $u,, v text{ independent} iff operatorname{cov}(u,v) = 0$.



            $$ operatorname{cov} left( frac{X Y}{sqrt{X^2 + Y^2}},frac{X^2 - Y^2}{sqrt{X^2 + Y^2}} right)$$
            $$ = operatorname{cov} left( frac{X Y}{sqrt{X^2 + Y^2}},frac{X^2}{sqrt{X^2 + Y^2}} right) - operatorname{cov} left( frac{X Y}{sqrt{X^2 + Y^2}},frac{Y^2}{sqrt{X^2 + Y^2}} right)$$



            Now make use of the symmetry of the expressions we have:
            $$ operatorname{cov}(cdots) = 0 $$



            To make this more rigorous one may rewrite the covariance as follows:
            $$ operatorname{cov} left( frac{X Y}{sqrt{X^2 + Y^2}},frac{X^2}{sqrt{X^2 + Y^2}} right) - operatorname{cov} left( frac{X Y}{sqrt{X^2 + Y^2}},frac{Y^2}{sqrt{X^2 + Y^2}} right) $$
            $$ = E left( frac{X^3 Y}{X^2 + Y^2} right) - E left( frac{X Y^3}{X^2 + Y^2} right) $$
            Now renaming $X to Y, , Y to X$ under the first expectation sign (note that this is the same as renaming variables under integral) we get the result.



            Covariance method can be carried even further, following the last line and the trick $ X = -X $ ($X$ is symmetric distribution) which we apply to the second $E$:
            $$ = E left( frac{X^3 Y}{X^2 + Y^2} right) - E left( frac{X Y^3}{X^2 + Y^2} right) = E left( frac{X^3 Y}{X^2 + Y^2} right) + E left( frac{X Y^3}{X^2 + Y^2} right) $$
            $$ = E left( frac{X Y(X^2 + Y^2)}{X^2 + Y^2} right) = E(XY) = 0$$



            The last equation is due to independence of $X, Y$ $implies 0=operatorname{cov}(X,Y)=E(XY) - E(X)E(Y)=E(XY).$







            share|cite|improve this answer














            share|cite|improve this answer



            share|cite|improve this answer








            edited Jan 6 at 15:22

























            answered Feb 24 '18 at 8:47









            Al PrihodkoAl Prihodko

            436




            436








            • 3




              $begingroup$
              There is an error in the first sentence here. Being Gaussian and having zero covariance does NOT imply independnce. It does imply independence if they are JOINTLY Gaussian. But you haven't mentioned that.
              $endgroup$
              – Michael Hardy
              Aug 20 '18 at 20:43














            • 3




              $begingroup$
              There is an error in the first sentence here. Being Gaussian and having zero covariance does NOT imply independnce. It does imply independence if they are JOINTLY Gaussian. But you haven't mentioned that.
              $endgroup$
              – Michael Hardy
              Aug 20 '18 at 20:43








            3




            3




            $begingroup$
            There is an error in the first sentence here. Being Gaussian and having zero covariance does NOT imply independnce. It does imply independence if they are JOINTLY Gaussian. But you haven't mentioned that.
            $endgroup$
            – Michael Hardy
            Aug 20 '18 at 20:43




            $begingroup$
            There is an error in the first sentence here. Being Gaussian and having zero covariance does NOT imply independnce. It does imply independence if they are JOINTLY Gaussian. But you haven't mentioned that.
            $endgroup$
            – Michael Hardy
            Aug 20 '18 at 20:43


















            draft saved

            draft discarded




















































            Thanks for contributing an answer to Mathematics Stack Exchange!


            • Please be sure to answer the question. Provide details and share your research!

            But avoid



            • Asking for help, clarification, or responding to other answers.

            • Making statements based on opinion; back them up with references or personal experience.


            Use MathJax to format equations. MathJax reference.


            To learn more, see our tips on writing great answers.




            draft saved


            draft discarded














            StackExchange.ready(
            function () {
            StackExchange.openid.initPostLogin('.new-post-login', 'https%3a%2f%2fmath.stackexchange.com%2fquestions%2f2647442%2fgiven-that-x-y-are-independent-n0-1-show-that-fracxy-sqrtx2y2%23new-answer', 'question_page');
            }
            );

            Post as a guest















            Required, but never shown





















































            Required, but never shown














            Required, but never shown












            Required, but never shown







            Required, but never shown

































            Required, but never shown














            Required, but never shown












            Required, but never shown







            Required, but never shown







            Popular posts from this blog

            Can a sorcerer learn a 5th-level spell early by creating spell slots using the Font of Magic feature?

            ts Property 'filter' does not exist on type '{}'

            mat-slide-toggle shouldn't change it's state when I click cancel in confirmation window